Search found 249 matches


I am not sure if "OA" is correct. Because in choice "A" which seems to refer back to "Elk" living in Rocky Mountains and per MGMAT and other grammar rules "Which" when used as pronoun should refer mack to immediate noun. Moreover "they" in non underl...

by amitabhprasad

Fri Jan 23, 2009 10:08 am
Forum: Sentence Correction
Topic: Elk now live
Replies: 2
Views: 2380

The reason I will filter out "A" was... If you take The proposed health care as doing multiple things then we need "and" between health insurance, and establish C) to establish can not be a modifier of a clause D) & E because it uses conjunction "for" and for needs ...

by amitabhprasad

Fri Jan 23, 2009 9:57 am
Forum: Sentence Correction
Topic: tense?
Replies: 3
Views: 1573

As per latest MGMAT Page 198
Not Also ... But ... construction
is used by GMAT in correct answer choice.
I was looking for an example, now have one.
More over you look into the choices only 'C' maintains parallelism.

by amitabhprasad

Thu Jan 22, 2009 10:53 pm
Forum: Sentence Correction
Topic: GMAT Prep SC
Replies: 10
Views: 2279

Unless we know the source.
IMO we shouldn't take OA for granted.
And I am finding hard to justify "B"

by amitabhprasad

Wed Jan 21, 2009 10:15 pm
Forum: Critical Reasoning
Topic: interest rates
Replies: 17
Views: 3751

can any body suggests why use of "instead of" in correct in "B"

by amitabhprasad

Wed Jan 21, 2009 10:12 pm
Forum: Sentence Correction
Topic: travelers
Replies: 6
Views: 1871

krisraam wrote:B.

I CAN'T THINK HOW I COULD BE "D"
I believe premise states if interest rates falls significantly then house price must rise
But as per "B" for any drop in interest rates housing price must fall. I am unable to justify this
Any other view ?

by amitabhprasad

Wed Jan 21, 2009 3:42 pm
Forum: Critical Reasoning
Topic: interest rates
Replies: 17
Views: 3751

I see it ow how it should be D.
9*3 =27
1/2*6 = 3
27*3 = 81
since when odd we should divide by 3 to keep it inside the CAGE
So it should be 27 and not 81
Got stumped ...

by amitabhprasad

Wed Jan 21, 2009 10:18 am
Forum: Problem Solving
Topic: GMATPRep weird m
Replies: 5
Views: 1584

One problem I see with 'B" is parallelism is not maintained. A) Modifier subject problem B) Parallelism . which is an … and scarcely it should be which is ... and which ... C) use of “and” it seems to imply the domestic cat has been scarcely sufficient… D) its genetic evolution that has scarcel...

by amitabhprasad

Wed Jan 21, 2009 10:09 am
Forum: Sentence Correction
Topic: the cats
Replies: 9
Views: 1993

IMO B the stem is 'what Mark is interpreting from Ronald's statement'. Per Ronald if the interest rates go down, house prices go up D. it would have been the correct answer if the stem was 'what can be inferred from this conversation' Isn't Mark also talking about significant interest rate drop ? n...

by amitabhprasad

Wed Jan 21, 2009 8:54 am
Forum: Critical Reasoning
Topic: interest rates
Replies: 17
Views: 3751

amitabh can you please explain in detail ..... Sure, a1 = 2, a2 = -3, a3 = 5, a4 = -1 ==> a1+a2+a3+a4 = 3 now we have an = an-4 ==> a5 = a5-4 = a1=2 a6 = a6-4 =a2 = -3 a7 = a7-4 = a3 = 5 a8 = a8-4 = a4 = -1 and so on you will see we get sequence in the block of 4 We are supposed to find the sum of ...

by amitabhprasad

Wed Jan 21, 2009 8:52 am
Forum: Problem Solving
Topic: trouble with sequence
Replies: 4
Views: 1208

I am getting answer as 74 ?

by amitabhprasad

Wed Jan 21, 2009 8:35 am
Forum: Problem Solving
Topic: trouble with sequence
Replies: 4
Views: 1208

For number to be terminating decimals. All the factors of denominators should be multiples of 2 or 5 i.e. check if the fraction can be reduced such that denominators only have multiples of 2 and/or 5. Go by this rule you will see all the denominators other then "E" has not only 2 but some ...

by amitabhprasad

Wed Jan 21, 2009 8:30 am
Forum: Problem Solving
Topic: any shortcut to a fraction quest??
Replies: 2
Views: 1117

My pick will be "D" as well.
Not sure how OA could be "B" when premise is clearly talking about significant interest drop ???

by amitabhprasad

Wed Jan 21, 2009 7:42 am
Forum: Critical Reasoning
Topic: interest rates
Replies: 17
Views: 3751

IMO A. D & E - Eliminated because of improper Subject. Between A and C. I eliminated C because it is wordy. @amitabhprasad, Can you explain in detail about the ans choice C? Regards, Karthi Hi Karthi, For the use of "had" please see my earlier post. Let me know if you need any more cl...

by amitabhprasad

Wed Jan 21, 2009 7:36 am
Forum: Sentence Correction
Topic: Scholars who once
Replies: 4
Views: 5890

IMO A since past perfet tense needs to be used for two events one preceding the another.. Once you have "before", "after" or any such prepositions indicating order of those events we don't need had. I think its important to understand the reason for using past perfect. We need p...

by amitabhprasad

Wed Jan 21, 2009 7:34 am
Forum: Sentence Correction
Topic: of the most popular resort destinations
Replies: 17
Views: 3143